Đến nội dung

Idie9xx nội dung

Có 313 mục bởi Idie9xx (Tìm giới hạn từ 06-05-2020)



Sắp theo                Sắp xếp  

#544520 CMR trong dãy số $m,f(m),f\left ( f(m) \right ),...$ co...

Đã gửi bởi Idie9xx on 16-02-2015 - 19:03 trong Phương trình hàm

xét hàm số $f(n)=\left [ n+\sqrt{n} \right ] \ \ \forall n\in \mathbb{N}^*$.Cho $m\geq 1$ là số tự nhiên.Xét dãy các số $m,f(m),f\left ( f(m) \right ),...$

CMR trong dãy số có vô hạn số chính phương 

 

U-Th

1. Xét các số nguyên dương $n$ có dạng $t^2+a$ với $t\geq a>0$ ta có

$f(t^2+a)=[t^2+a+\sqrt{t^2+a}]= t^2+t+a$

$f(t^2+t+a)=[t^2+t+a+\sqrt{t^2+t+a}]=t^2+2t+a=(t+1)^2+(a-1)$

Từ đó ta dễ dàng chứng minh được $f^{(2a)}(t^2+a)=(t+a)^2$ ($f^{(k)}(n)=f(f(f(...(n)...)))$ có $k$ lần $(..)$)

2. Xét các số nguyên dương $n$ có dạng $t^2-a$ với $t>a\geq 0$ ta có

$f(t^2-a)=[t^2+a+\sqrt{t^2-a}]=t^2+t-a-1$

Mà $t^2+t-a-1$ thuộc dạng 1 rồi nên ta chứng minh được $f^{(2(t-a)-1)}(t^2-a)=(2t-a-1)^2$

Từ các trường hợp 1 và 2 ta rút ra nhận xét với mỗi số nguyên dương $m$ ta luôn tìm được các số nguyên dương $k,p$ thỏa $f^{(k)}(m)=p^2$ từ đó chứng minh được yêu cầu của đề bài :)




#544503 CMR $f(n )=g(n)\ \ \forall n\in \mathbb{N...

Đã gửi bởi Idie9xx on 16-02-2015 - 17:59 trong Phương trình hàm

Cho $f:\mathbb{N}^*\rightarrow \mathbb{N}^*$ là toàn ánh và $g:\mathbb{N}^*\rightarrow \mathbb{N}^*$ đơn ánh thỏa mãn $f(n)\geq g(n)\ \ \forall n\in \mathbb{N}^*$

CMR $f(n )=g(n)\ \ \forall n\in \mathbb{N}^*$

 

U-Th

Giả sử tồn tại số $m_0$ thỏa $f(m_0)>g(m_0)$

Do $f$ toàn ánh nên tồn tại một số $m_1$ thỏa $f(m_1)=g(m_0)$

Mà $f(m_1)\geq g(m_1)\Rightarrow g(m_0)>g(m_1)$ ( do $g$ đơn ánh)

Dễ thấy sẽ có một dãy vô hạn $(m_t)$ thỏa $g(m_0)>g(m_1)>...>g(m_{t-1})>g(m_t)>...$

Điều này là vô lí bởi dãy $g(m_t)$ bị chặn. Nên ta có $f(n)=g(n),\forall n \in \mathbb{N}^*$ đpcm




#544502 $f\left ( \dfrac{y}{f(x+1)} \right )+...

Đã gửi bởi Idie9xx on 16-02-2015 - 17:52 trong Phương trình hàm

Giả sử tồn tại $z\in R^{+}$  sao cho :   $f(z)-\frac{1}{z}\neq 0$   

 

Thay $x\rightarrow \frac{1}{y(f(y))-1}$   ta được :   

 

              $f(\frac{y}{f(\frac{yf(y)}{yf(y)-1})})+f(y)=f(y)$  

 

              $\Rightarrow f(\frac{y}{f(\frac{yf(y)}{yf(y)-1})})=0$   ( vô lý )

 

Nên $f(x)=\frac{1}{x}$ với mọi  $x\in R^{+}$

 

P/s : Chẳng biết đúng hay sai !!! :icon6: 

Theo cách này ta chứng minh được $xf(x)\leq 1$ hay $f(x)\leq \frac{1}{x}$

$\Rightarrow f(y)=f(\frac{y}{f(x+1)})+f(\frac{x+1}{xf(y)})\leq \frac{f(x+1)}{y}+\frac{xf(y)}{x+1}$

$\Rightarrow yf(y)\leq (x+1)f(x+1)$

$\Rightarrow xf(x)=c, \forall x\geq 1$ dễ dàng tìm được $c=1$

$x\leq 1$ không biết xử lí sao :))




#535214 $f(x-f(y))=2f(x)+x+f(y)$

Đã gửi bởi Idie9xx on 28-11-2014 - 19:56 trong Phương trình hàm

Tìm hàm số $f: \mathbb{R} \to \mathbb{R}$ thỏa mãn

$f(x-f(y))=2f(x)+x+f(y)$

Cho $x=f(y)$ ta có $f(0)=2(f(f(x))+f(x))$ thay $x=0$ có $2f(f(0))=-f(0)$

Thay $x,y$ lần lượt bằng $x+f(x),x$ ta có $2f(f(x)+x)=-(f(x)+x)$

Từ hai điều trên thay $x$ bằng $f(x)\Rightarrow f(\frac{f(0)}{2})=-\frac{f(0)}{4}$

Cho $x=0,y=f(0)$ có $f(-f(f(0)))=2f(0)+f(f(0))\Rightarrow f(\frac{f(0)}{2})=2f(0)-\frac{f(0)}{2}$

$\Rightarrow -\frac{f(0)}{4}=2f(0)-\frac{f(0)}{2}\Rightarrow f(0)=0$

Cho $y=0\Rightarrow f(x-f(0))=2f(x)+x+f(0)\Rightarrow f(x)=-x$

Vậy hàm thỏa mãn đề bài là $f(x)=-x$ :))




#534327 $f(2x-f(x))=x$

Đã gửi bởi Idie9xx on 23-11-2014 - 10:20 trong Phương trình hàm

Vậy mình có thể hỏi là lời giải cuối cùng của bài này là như thế nào vậy ạ. :)

Đã chỉnh sửa ở trên. Bạn có thắc mắc gì nữa không :)




#534305 $f(2x-f(x))=x$

Đã gửi bởi Idie9xx on 23-11-2014 - 07:37 trong Phương trình hàm

Chỗ $a+nc=a'+m'c$ , vì $a,c,a',c'$ là số thực thì sao lại luôn có thể tìm được các số nguyên $m,n$ thỏa mãn đẳng thức đó vậy ? mà câu ( do hàm liên tục có thể chỉ sấp sỉ nhau nghĩa là gì vậy ?)

 

 

cái kĩ thuật mà tạo ra được $f(b)-b=c$ là gì vậy, và khi nào thì áp dụng được nó vậy @@

Do lỗi kĩ thuật rồi :v em nhờ hồi trước có dùng một cái tương tự này nên áp dụng xem thế nào ai ngờ sai ~.~ thực ra nên dùng cái chứng minh $f$ là hàm tăng kia :))




#534304 CMR không tồn tại hàm $f$ và $g$ thỏa $f\l...

Đã gửi bởi Idie9xx on 23-11-2014 - 07:19 trong Phương trình hàm

Có phải là cho $x,y$ phân biệt và bé xíu thì phần nguyên của nó đều bằng 0 phải không, và khi cộng vào t thì không làm thay đổi phần nguyên của t ?

Bé xíu như $0,02$ thì phần nguyên bằng $0$, khi cộng $t$ vào thì cũng tùy theo $t$ mà 2 phần nguyên của nó có bằng nhau hay hơn kém nhau 1 đơn vị :)




#534275 $f(2x-f(x))=x$

Đã gửi bởi Idie9xx on 22-11-2014 - 21:40 trong Phương trình hàm

Tìm hàm liên tục $f: R\rightarrow R$  thoả mãn:
  1) $f$ là đơn ánh

  2) $f(2x-f(x))=x$

  3) Tồn tại $x_{0}$ thỏa mãn $f(x_{0})=x_{0}$

Từ $2)$ ta thấy $f$ toàn ánh, kết hợp với đơn ánh và liên tục nên $f$ đơn điệu.

Giả sử hàm là hàm giảm ta có: $x>y \Rightarrow f(2x-f(x))>f(2y-f(y))$

$\Rightarrow 2x-f(x)<2y-f(y) \Rightarrow f(x)-f(y)>2(x-y)>0 \Rightarrow f(x)>f(y)$ vô lí.

Nên $f$ là hàm tăng. $(*)$

Ta có: $f(a+(a-f(a)))=a$ Giả sử $a-f(a)=c\neq 0$ thì $f(a+c)=a$

Với $b=a+c$ thì $b-f(b)=c$

Vậy ta có $f(a+nc)=a+(n-1)c$

Nếu $a>x_0$ mà $c<0$ thì tồn tại $n$ thỏa $a+nc<x_0$ và $a+(n-1)c>x_0$

Khi đó $f(a+nc)<f(x_0)=x_0<a+(n-1)c$ mâu thuẫn.

$\Rightarrow a>x_0$ thì $c> 0$

Giả sử ta có $a'$ thỏa $a'-f(a')=c'\neq 0,c'< c$ thì tồn tại 2 số $m,n$ nguyên dương thỏa:

$a+nc>a'+mc'$ mà $a+(n-1)c<a'+(m-1)c' \Rightarrow f(a+nc)<f(a'+mc')$ không thỏa mãn $(*)$

Chứng minh tương tự nếu có $c'>c$

Nên $x-f(x)=c, \forall x>x_0$ dễ thấy tồn tại $x$ thỏa $x-c<x_0$ không thỏa mãn $(*)$

$\Rightarrow f(x)=x, \forall x>x_0$

Chứng minh tương tự với $x<x_0$ ta có được $f(x)=x, \forall x<x_0$

Kết luận hàm thỏa là $f(x)=x$ 

Ps: $a+nc>a'+mc'$ mà $a+(n-1)c<a'+(m-1)c'$ có nghĩa là $c-c'>(a+nc)-(a'+mc')>0$

Không biết có còn sai chỗ nào không :wacko:




#534231 Tìm hàm liên tục $f(f(x)+y)=2x+f(y)$

Đã gửi bởi Idie9xx on 22-11-2014 - 19:23 trong Phương trình hàm

Mình thấy pt cauchy có dạng f(x)+f(y)=f(x+y) đúng với mọi x,y thuộc R, nhưng ở đây m,n là thuộc tập giá trị của ánh xạ f, chưa chạy toàn R thì cũng được áp dụng à, bạn giải thích rõ với, mình không hiểu lắm.

Do $f(f(x))=2x$ nên $f$ toàn ánh. Bài giải trên thiếu cái này.




#533829 CMR không tồn tại hàm $f$ và $g$ thỏa $f\l...

Đã gửi bởi Idie9xx on 19-11-2014 - 21:51 trong Phương trình hàm

chứng minh rằng không tồn tại cặp hàm số $f:\mathbb{R}\rightarrow \mathbb{R}$ và $g:\mathbb{R}\rightarrow \mathbb{R}$ thỏa mãn đồng thời các điều kiện sau

$i)$  $g$ tuần hoàn

$ii)$ $f\left ( \left \lfloor x \right \rfloor \right )+g(x)=x^3$

 

NTP

Giả sử hàm $g(x)$ tuần hoàn chu kỳ $t$ ta có:

$f\left ( \left \lfloor x+t \right \rfloor \right )-f\left ( \left \lfloor x \right \rfloor \right )=(x+t)^3-x^3=t(x^2+xt+t^2)$

Ta thấy tồn tại 2 số $x,y$ phân biệt thỏa $\left \lfloor x \right \rfloor=\left \lfloor y \right \rfloor,\left \lfloor x+t \right \rfloor=\left \lfloor y+t \right \rfloor,x+y+t \neq 0$

Ta có $f\left ( \left \lfloor x+t \right \rfloor \right )-f\left ( \left \lfloor x \right \rfloor \right )=t(x^2+xt+t^2)$

và $f\left ( \left \lfloor y+t \right \rfloor \right )-f\left ( \left \lfloor y \right \rfloor \right )=t(y^2+yt+t^2)$

Do $\left \lfloor x \right \rfloor=\left \lfloor y \right \rfloor,\left \lfloor x+t \right \rfloor=\left \lfloor y+t \right \rfloor$

nên $\Rightarrow t(x^2+xt+t^2)=t(y^2+yt+t^2) \Rightarrow t(x-y)(x+y+t)=0 \Rightarrow t=0$ vô lí

Nên không tồn tại cặp hàm $f,g$ thỏa mãn (dpcm) >:)




#533814 Biết $ \sum_{1}^{\infty }\frac{1...

Đã gửi bởi Idie9xx on 19-11-2014 - 21:06 trong Giải tích

$ \sum_{1}^{\infty }\frac{1}{ n^2 } =\frac{ \pi^2 }{6}$ Tính $ \sum_{1}^{\infty }\frac{ 1 }{ (2n-1)^2 } $

$S=\sum_{1}^{\infty }\frac{1}{ n^2 } =\frac{ \pi^2 }{6}$

$T=\sum_{1}^{\infty }\frac{ 1 }{ (2n-1)^2 }$

Ta có $\dfrac{S}{4}=\sum_{1}^{\infty }\frac{ 1 }{ (2n)^2 }$

$\Rightarrow T+\dfrac{S}{4}=\sum_{1}^{\infty }\frac{ 1 }{ (2n-1)^2 }+\sum_{1}^{\infty }\frac{ 1 }{ (2n)^2 }=\sum_{1}^{\infty }\frac{1}{ n^2 }=S$

$\Rightarrow T=\dfrac{3}{4}S=\dfrac{\pi^2}{8}$ :lol:




#511732 $f\left ( f\left ( m \right ) \right ) +f\left...

Đã gửi bởi Idie9xx on 08-07-2014 - 20:35 trong Phương trình hàm

Tìm tất cả các hàm đơn ánh $f:N\rightarrow R$ thỏa:

    $\left\{\begin{matrix} f\left ( 1 \right )=2;f\left ( 2 \right )=4\\f\left ( f\left ( m \right )+f\left ( n \right ) \right )=f\left ( f\left ( m \right ) \right ) +f\left ( n \right ) \end{matrix}\right.$

Cho $m=x,n=1\Rightarrow f(f(x)+f(1))=f(f(x))+f(1)=f(f(x))+2$

Cho $m=1,n=x\Rightarrow f(f(1)+f(x))=f(f(1))+f(x)=f(2)+f(x)=f(x)+4$

Từ 2 cái trên $\Rightarrow f(f(x))+2=f(x)+4\Rightarrow f(f(x))=f(x)+2$

Do là hàm đơn ánh nên ta có thể suy ra hàm cần tìm là $f(n)=n+2$ :P




#484444 $f(x^2+f(y))=y+f^2(x),\;\;\forall x,y\in \mathb...

Đã gửi bởi Idie9xx on 23-02-2014 - 20:12 trong Phương trình hàm

Còn cái bài hàm quen thuộc mà anh nói, anh có thể làm hoặc hướng dẫn em được không ạ ?

$f(x^2)=(f(x))^2\geq 0$ nên với $x> 0 \Rightarrow f(x)\geq 0$

Cho $x>y$ ta có

$f(x)=f(y)+f(x-y)\geq f(y)$, vậy nên $f$ là hàm tăng.

$f(x+y)=f(x)+f(y)\Rightarrow f(x)=cx,\forall x\in \mathbb{Q}$ thử lại thấy $c=1$ thoả.

Với mỗi số vô tỉ $r$ luôn tồn tại hai dãy số hữu tỉ $(p_n),(q_n)$ thoả

$\lim_{n\rightarrow +\infty} q_n=\lim_{n\rightarrow +\infty} p_n=r$ và $q_n>r>p_n,\forall n \in \mathbb{N}$

Ta có $f(q_n)\geq f(r)\geq f(p_n)$ khi $n\rightarrow +\infty \Rightarrow f(r)=r$

$\Rightarrow f(x)=x,\forall x\in \mathbb{R}$ :))




#484387 $f(x^2+f(y))=y+f^2(x),\;\;\forall x,y\in \mathb...

Đã gửi bởi Idie9xx on 23-02-2014 - 16:33 trong Phương trình hàm

Tìm tất cả các hàm số $f:\mathbb{R}\rightarrow \mathbb{R}$ và thỏa mãn :

$$f(x^2+f(y))=y+f^2(x),\;\;\forall x,y\in \mathbb{R}\;\;\;\;(1)$$

 

Mình chứng minh được $\left\{\begin{matrix} f(x^2)=f^2(x),\;\forall x\in \mathbb{R}\\ f(x+y)=f(x)+f(y),\;\;x,y\in \mathbb{R} \end{matrix}\right.$

Sau đó tính biểu thức $f((x+1)^2)$ theo hai cách thì nó ra thế này $2f(x)(1-f(1))=f^2(1)$ ????

Rõ ràng hàm $f(x)=x$ thỏa $(1)$. Không biết sai sai chỗ nào @@.

....

Ta sẽ tính biểu thức $f((x+1)^2)$ theo $(2)$ và $(3)$ :

$f((x+1)^2)=f^2(x+1)=(f(x)+f(1))^2=f^2(x)+2f(x)f(1)+f^2(1),\;\forall x\in \mathbb{R}$

$f((x+1)^2)=f(x^2+2x+1)=f(x^2)+2f(x)+f(1)=f^2(x)+2f(x)+f(1),\;\forall x\in \mathbb{R}$

Từ hai kết quả này ta suy ra :

$f^2(x)+2f(x)f(1)+f^2(1)=f^2(x)+2f(x)+f(1),\;\forall x\in \mathbb{R}\Rightarrow 2f(x)(1-f(1))=f^2(1)\;\;(*)$ [/hide] 

 

Ai có cách nào hay thì làm luôn nhé ! 

Từ $(2)$ và $(3)$ có thể suy ra luôn là $f(x)=x$ rồi mà :P ( một bài hàm quen thuộc :closedeyes:  )

Còn cái $(*)$ là do viết thiếu, phải là $f^2(1)-f(1)$ :lol:  bạn trừ lại thử xem :)




#483393 $$f(f(f(n)))= f(n+1)+1$$

Đã gửi bởi Idie9xx on 16-02-2014 - 09:21 trong Phương trình hàm

Bài toán :

Tìm hàm số $f \, :\, \mathbb{N}\to \mathbb{N}$ sao cho : $$f(f(f(n)))= f(n+1)+1\,\,\forall n\in\mathbb{N}$$

Dễ thấy với mỗi số tự nhiên $k$ luôn tồn tại 1 số tự nhiên $m$ thỏa $f(m)=f(k)+1$ :ohmy:

Bằng qui nạp ta chứng minh được rằng với mỗi số tự nhiên $t>f(k)$ thì tồn tại số tự nhiên $u$ sao cho $f(u)=t$ :lol:

Giả sử ta có số $a$ sao cho $f(a)=0$ thì tồn tại số tự nhiên $b$ sao cho $f(f(b))=a\Rightarrow f(f(f(b)))=0\Rightarrow f(b+1)=-1$ (mâu thuẫn)

Nên không tồn tại $a$ thỏa $f(a)=0$

Giả sử ta có số $a$ sao cho $f(a)=1$ thì tồn tại số tự nhiên $b$ sao cho $f(f(b))=a\Rightarrow f(f(f(b)))=1\Rightarrow f(b+1)=0$ (mâu thuẫn)

Nên không tồn tại $a$ thỏa $f(a)=1$

Chứng minh qui nạp với điều giả sử tương tự ta thấy không có hàm thỏa đề :mellow:

Vậy không có hàm thỏa mãn đề bài :ukliam2:  :))

-----------------

WS : Với hàm $f(n)=n+1$ thì sao a ?




#480814 $f\left ( x+\dfrac{y}{x} \right )=f(x...

Đã gửi bởi Idie9xx on 04-02-2014 - 12:54 trong Phương trình hàm

Tìm hàm số $f:\mathbb{Q}^+\rightarrow \mathbb{Q}^+$ thỏa mãn :

$$f\left ( x+\dfrac{y}{x} \right )=f(x)+\dfrac{f(y)}{f(x)}+2y$$

Giả sử $f$ là hàm hằng thì ta thấy không thỏa.

Cho $P(x,y)$ có tính chất $f\left ( x+\dfrac{y}{x} \right )=f(x)+\dfrac{f(y)}{f(x)}+2y$

$P(x,x)\Rightarrow f(x+1)=f(x)+2x+1\Rightarrow f(x+1)-(x+)^2=f(x)-x^2$

Bằng qui nạp ta chứng minh được $f(x+n)-(x+n)^2=f(x)-x^2$

$P(1,x)\Rightarrow f(x+1)=f(1)+\dfrac{f(x)}{f(1)}+2x$

$\Rightarrow f(x)+1=f(1)+\dfrac{f(x)}{f(1)}\Rightarrow f(x)=\dfrac{f(1)-1}{1-\dfrac{1}{f(1)}$

$\Rightarrow f(1)=1\Rightarrow f(n)=n^2,\forall n\in \mathbb{Z^+}$

Viết $x$ dưới dạng $x=\dfrac{p}{q};p,q\in \mathbb{Z^+}$

$P(q,p)\Rightarrow f(q+x)=f(q)+\dfrac{f(p)}{f(q)}+2p=(q+\dfrac{p}{q})^2$

$\Rightarrow f(x)=(\dfrac{p}{q})^2=x^2,\forall x\in \mathbb{Q^+}$ :))




#477672 CMR: $f(3)=g(3)$

Đã gửi bởi Idie9xx on 17-01-2014 - 14:12 trong Phương trình hàm

Cho hàm $f,g$ :$(2;4)\rightarrow (2;4)$ thỏa mãn: $\left\{\begin{matrix} f(g(x))=g(f(x)) & & \\ f(x) g(x)=x^{2}& & \end{matrix}\right.$. CMR:

$f(3)=g(3)$

$f(g(x))=g(f(x))\Rightarrow \dfrac{(g(x))^2}{g(g(x))}=\dfrac{(f(x))^2}{f(f(x))}\Rightarrow \dfrac{f(f(x))}{g(g(x))}=\left(\dfrac{f(x)}{g(x)} \right)^2$

Bằng qui nạp ta chứng minh được:

$\dfrac{f_{n+1}(x)}{g_{n+1}(x)}=\left(\dfrac{f_n(x)}{g_n(x)} \right)^2=...=\left(\dfrac{f(x)}{g(x)} \right)^{2^n},n\in \mathbb{N^*}$

Mà do $\dfrac{1}{2}<\dfrac{f_{n+1}(x)}{g_{n+1}(x)}<2,\dfrac{1}{2}<\dfrac{f(x)}{g(x)}<2$ nên khi cho $n\rightarrow +\infty$ thì:

 - Nếu $\dfrac{f(x)}{g(x)}>1\Rightarrow \dfrac{f_{n+1}(x)}{g_{n+1}(x)}\rightarrow +\infty$ mâu thuẫn.

 - Nếu $\dfrac{f(x)}{g(x)}<1\Rightarrow \dfrac{f_{n+1}(x)}{g_{n+1}(x)}\rightarrow 0$ mâu thuẫn.

 - Nếu $\dfrac{f(x)}{g(x)}=1$ hay $f(x)=g(x)$ thỏa mãn.

Từ đây suy ra $f(3)=g(3)$ (dpcm). :))




#477670 $f\left ( f\left ( y+f(x) \right ) \right )=f(x+y)+f...

Đã gửi bởi Idie9xx on 17-01-2014 - 14:00 trong Phương trình hàm

Anh giải thích giùm em được không ạ ? Em mới học PTH nên cũng không chắc nữa ~~

 

Trong $(2)$ thay $y$ bởi $x$ và $f(x)$ bởi $y$ : 

$f\left ( f\left ( y+f(x) \right ) \right )=f\left ( x+f(x) \right )+y+f(x),\;\forall x,y\in \mathbb{R}\;\;(3)$

Trong $(4)$ thay $x=y$ : $f(x+f(x))+x=f(x+f(x))+f(x),\;\forall x\in \mathbb{R}\Leftrightarrow f(x)=x,\;\forall x\in \mathbb{R}$

Thay $f(x)$ bởi $y$ với thay $y$ bởi $x$ thì $y=f(x)$ rồi, mà đoạn kia lại thay $x=y$ làm sao được :)) nghĩa là tự cho luôn $f(x)=x$ rồi còn gì :luoi:




#477632 $f\left ( f\left ( y+f(x) \right ) \right )=f(x+y)+f...

Đã gửi bởi Idie9xx on 16-01-2014 - 22:39 trong Phương trình hàm

Không biết đúng không ~~

$$f\left ( f\left ( y+f(x) \right ) \right )=f(x+y)+f(x)+y,\;\;\forall x,y\in \mathbb{R}\;\;\;\;(1)$$

Trong $(1)$ thay $y$ bởi $f(y)$ : $f\left ( f\left ( f(x)+f(y) \right ) \right )=f(x+f(y))+f(x)+f(y),\forall x,y\in \mathbb{R}\;\;(2)$

Trong $(2)$ thay $y$ bởi $x$ và $f(x)$ bởi $y$ : 

$f\left ( f\left ( y+f(x) \right ) \right )=f\left ( x+f(x) \right )+y+f(x),\;\forall x,y\in \mathbb{R}\;\;(3)$

Từ $(2)(3)$ được $f\left ( x+f(x) \right )+y+f(x)=f(x+f(y))+f(x)+f(y),\;\forall x,y\in \mathbb{R}\Leftrightarrow f(x+f(x))+y=f(x+f(y))+f(y),\;\forall x,y\in \mathbb{R}\;\;(4)$

Trong $(4)$ thay $x=y$ : $f(x+f(x))+x=f(x+f(x))+f(x),\;\forall x\in \mathbb{R}\Leftrightarrow f(x)=x,\;\forall x\in \mathbb{R}$

Thử lại không thỏa. Vậy không tồn tại hàm số thỏa đề.

Sao có thể thay $f(x)$ bởi $y$ được :) và $(3)$ chắc là sai.

P/s: Thử lại quên không nhìn đề :))




#477594 $f\left ( f\left ( y+f(x) \right ) \right )=f(x+y)+f...

Đã gửi bởi Idie9xx on 16-01-2014 - 20:05 trong Phương trình hàm

Tìm các hàm số $f:\mathbb{R}\rightarrow \mathbb{R}$ thỏa mãn :

$$f\left ( f\left ( y+f(x) \right ) \right )=f(x+y)+f(x)+y,\;\;\forall x,y\in \mathbb{R}$$

Với $P(x,y)$ có tính chất $f\left ( f\left ( y+f(x) \right ) \right )=f(x+y)+f(x)+y$

$P(x,f(0))\Rightarrow f(f(f(0)+f(x)))=f(x+f(0))+f(x)+f(0)$

$P(0,f(x))\Rightarrow f(f(f(x)+f(0)))=f(f(x))+f(0)+f(x)$

$\Rightarrow f(x+f(0))=f(f(x)),(1)$

$P(0,x)\Rightarrow f(f(x+f(0))=f(x)+f(0)+x,(2)$

$P(x,0)\Rightarrow f(f(f(x))=f(x)+f(x),(3)$

Từ $(1),(2),(3)\Rightarrow f(x)=f(0)+x$

Thử lại không có hàm thỏa đề :P




#477205 $f(n)+f(n+1)=f(n+2)f(n+3)-1996$

Đã gửi bởi Idie9xx on 14-01-2014 - 14:36 trong Phương trình hàm

Tìm tất cả các hàm $f:N\rightarrow N$ thỏa mãn :

     $f(n)+f(n+1)=f(n+2)f(n+3)-1996$

Đọc thử bài này xem:

http://diendantoanho...ll-nin-mathbbn/




#477202 $f(f(x+y))=f(x+y)+f(x).f(y)-xy, \forall x,y \in \mathbb...

Đã gửi bởi Idie9xx on 14-01-2014 - 14:22 trong Phương trình hàm

vậy nếu không tồn tại a thi sao nhỉ ,co can cm f toàn ánh hay gì ko bạn  :icon6:

p/s: bạn có tài liệu nào về giải pt hàm bằng dãy số giới hạn không ,cho mình xin với 

 

Đọc tiếp phần sau thì mình đã chứng minh được có tồn tại $a$ thỏa mãn rồi ( $a=f(0)$ hoặc $a=-f(0)$ ) :)

Mình không có tại liệu đó đâu, toàn dựa vào tính chất dãy số mà làm thôi :))




#476349 $f(f(n-1))=f(n+1)-f(n)$

Đã gửi bởi Idie9xx on 09-01-2014 - 19:15 trong Phương trình hàm

 Có tồn tại hay không $f: N*\rightarrow N*$ thỏa: $$f(f(n-1))=f(n+1)-f(n)$$

....fix lại đề... :icon4: 

Vô đây xem lời giải của mình nhé :P

http://diendantoanho...75-ffn-1fn1-fn/




#476159 Giải pt hàm: $f(x^3+x) \leq x \leq (f(x))^3+f(x)$

Đã gửi bởi Idie9xx on 08-01-2014 - 19:41 trong Phương trình hàm

Một bài trên Crux.

Xác định hàm số f:

$f:\mathbb{R}\rightarrow \mathbb{R}$

sao cho $\forall x \in \mathbb{R}$ ta có:

$f(x^3+x) \leq x \leq (f(x))^3+f(x)$ 

$x\leq (f(x))^3+f(x),(1)$

Thay $x$ bằng $x^3+x$ vào phương trình $(1)$ có

$x^3+x\leq (f(x^3+x))^3+f(x^3+x)\leq x^3+x$

$\Rightarrow f(x^3+x)=x\Rightarrow x=(f(x))^3+f(x)$

Do hàm số $t^3+t$ luôn đồng biến nên với $x_0$ là nghiệm của phương trình $t^3+t=x$ ( ẩn $t$) thì ta có $f(x)=x_0$ :))
 




#474735 Chứng minh rằng $\frac{MA}{MB}+\frac{...

Đã gửi bởi Idie9xx on 02-01-2014 - 15:39 trong Hình học phẳng

Cho đường tròn (I) nội tiếp tam giác đều ABC. Một đường thẳng d tiếp xúc với (I) và cắt các cạnh AB, AC lần lượt tại M,N. Chứng minh rằng  

 

 $\frac{MA}{MB}+\frac{NA}{NC} = 1$

Với $F,D$ lần lượt là trung điểm của đoạn $AB,AC$

Đặt $\alpha=\widehat{FIM},\beta=\widehat{DIN}$ dễ thấy $\alpha+\beta=60^{\circ}$

Có $\tan\alpha=\dfrac{FM}{FI},\tan\beta=\dfrac{DN}{DI},\tan60^{\circ}=\dfrac{AF}{IF}=\dfrac{AD}{DI}$

Ta có $\frac{MA}{MB}+\frac{NA}{NC} = \frac{AF-MF}{AF+MF}+\frac{AD-DN}{AD+AN} $

$=\dfrac{\dfrac{AF}{FI}-\dfrac{MF}{FI}}{\dfrac{AF}{FI}+\dfrac{MF}{FI}}+\dfrac{\dfrac{AD}{DI}-\dfrac{ND}{DI}}{\dfrac{AD}{DI}+\dfrac{ND}{DI}}$

$=\dfrac{\tan60^{\circ}-\tan\alpha}{\tan60^{\circ}+\tan\alpha}+\dfrac{\tan60^{\circ}-\tan\beta}{\tan60^{\circ}+\tan\beta}=1$

$\Rightarrow \dfrac{\tan(\alpha+\beta)-\tan\alpha}{\tan(\alpha+\beta)+\tan\alpha}+\dfrac{\tan(\alpha+\beta)-\tan\beta}{\tan(\alpha+\beta)+\tan\beta}=1,(*)$

Cái $(*)$ bạn tự chứng minh xem có đúng không :) ( dựa vào công thức $\tan(\alpha+\beta)=\dfrac{\tan\alpha+\tan\beta}{1-\tan\alpha\cdot \tan\beta}$ )
58421163.mnl.png